0 Daumen
531 Aufrufe

Aufgabe:

$$ \sum _ { n = 0 } ^ { \infty } \frac { ( 3 n ) ! ( n ! ) ^ { 2 } } { ( 4 n ) ! \sqrt { ( 2 n ) ! } } x ^ { n } $$

Weiß jemand, wie man den Konvergenzradius bestimmt?

Avatar von

limn→∞ |an+1 / an| < 1

2 Antworten

0 Daumen

Wenn du an / an+1 bildest und kürzt, sind die Fakultäten weg und du hast im Zähler

(4n+1)(4n+2)(4n+3)(4n+4)* √ ((2n+1)(2n+2))

und im Nenner

(3n+1)(3n+2)(3n+3)(n+1)2

Wenn du jetzt noch durch n5 kürzt ( jede Klammer durch n) und in der Wurzel durch n2 dann

kannst du den Grenzwert für n gegen unendlich ablesen:

( 4 * 4 * 4 * 4 * √4  )    /   ( 3 * 3 * 3 * 1 * 1 ) = 512 / 27

Das ist der KOnv.rad.

Avatar von 288 k 🚀
0 Daumen

Hallo Fabi! :-)
$$  \frac{|a_{n+1}|}{|a_n|} = \frac{\frac{(3(n+1))!((n+1)!)^2 |x^{n+1}|}{(4(n+1))!\sqrt{(2(n+1))!}}}{\frac{(3n)!(n!)^2 |x^n|}{(4n)!\sqrt{(2n)!}}} = \\\frac{(3(n+1))!((n+1)!)^2 |x^{n+1}|} {(4(n+1))!\sqrt{(2(n+1))!}} \cdot \frac{(4n)!\sqrt{(2n)!}}{(3n)!(n!)^2 |x^n|} = \\\frac{(3(n+1))!}{(3n)!} \cdot \frac{(4n)!}{(4(n+1))!} \cdot \frac{\left((n+1)!  \right)^2}{\left((n)!\right)^2} \cdot \frac{\sqrt{(2n)!}}{\sqrt{(2(n+1))!}} \cdot \frac{|x^{n+1}|}{|x^n|} =\\\frac{(3n+1)(3n+2)(3n+3)}{1} \cdot \frac{1}{(4n+1)(4n+2)(4n+3)(4n+4)} \cdot \frac{(n+1)^2}{1} \cdot \frac{1}{\sqrt{(2n+1)(2n+2)}} \cdot \frac{|x|}{1} =\\\frac{(3n+1)(3n+2)(3n+3)}{(4n+1)(4n+2)} \cdot \frac{(n+1)^2}{(4n+3)(4n+4)} \cdot \frac{1}{\sqrt{(2n+1)(2n+2)}} \cdot |x| = \\\frac{(3n+1)(3n+2)(3n+3)}{(4n+1)(4n+2)} \cdot \frac{n^2+2n+1}{16n^2+28n+12} \cdot \frac{1}{\sqrt{4n^2+6n+2}} \cdot |x| = \\\frac{(3n+1)(3n+2)}{(4n+1)(4n+2)} \cdot \frac{n^2+2n+1}{16n^2+28n+12} \cdot \sqrt{ \frac{(3n+3)^2}{4n^2+6n+2}} \cdot |x| = \\\frac{(3n+1)(3n+2)}{(4n+1)(4n+2)} \cdot \frac{n^2+2n+1}{16n^2+28n+12} \cdot \sqrt{ \frac{9n^2+18n+9}{4n^2+6n+2}} \cdot |x| = \\\frac{(3+\frac{1}{n})(3+\frac{2}{n})}{(4+\frac{1}{n})(4+\frac{2}{n})} \cdot \frac{n^2(1 + \frac{2}{n}+\frac{1}{n^2})}{n^2(16+\frac{28}{n}+\frac{12}{n^2})} \cdot \sqrt{ \frac{n^2(9+\frac{18}{n}+\frac{9}{n^2})}{n^2(4+\frac{6}{n}+\frac{2}{n^2})}} \cdot |x| = \\\frac{(3+\frac{1}{n})(3+\frac{2}{n})}{(4+\frac{1}{n})(4+\frac{2}{n})} \cdot \frac{(1 + \frac{2}{n}+\frac{1}{n^2})}{(16+\frac{28}{n}+\frac{12}{n^2})} \cdot \sqrt{ \frac{(9+\frac{18}{n}+\frac{9}{n^2})}{(4+\frac{6}{n}+\frac{2}{n^2})}} \cdot |x| \\\lim_{n \to \infty}\frac{|a_{n+1}|}{|a_n|} = |x| \cdot \lim_{n \to \infty}\frac{(3+\frac{1}{n})(3+\frac{2}{n})}{(4+\frac{1}{n})(4+\frac{2}{n})} \cdot \frac{(1 + \frac{2}{n}+\frac{1}{n^2})}{(16+\frac{28}{n}+\frac{12}{n^2})} \cdot \sqrt{ \frac{(9+\frac{18}{n}+\frac{9}{n^2})}{(4+\frac{6}{n}+\frac{2}{n^2})}} = \\|x| \cdot \frac{3\cdot3}{4\cdot4} \cdot \frac{1}{16} \cdot \sqrt{ \frac{9}{4}} = |x| \cdot\frac{27}{512} \\ |x| \cdot \frac{27}{512} < 1 \Rightarrow |x| <  \frac{512}{27}$$

Beste Grüße

gorgar

Avatar von 11 k

Ein anderes Problem?

Stell deine Frage

Willkommen bei der Mathelounge! Stell deine Frage einfach und kostenlos

x
Made by a lovely community